You are on page 1of 14

University of Waterloo

Sample MATH 135 Final Examination


Term: Fall

Year: 2014

Provide the following identifying information. Please print legibly using CAPITAL letters.
Family name:

Given name:
USERNAME

SECTION

ID number:

Course abbreviation and number:

MATH 135

Course title:

Algebra for Honours Mathematics

Date of exam:

December 09, 2014

Exam period:

12:30 PM to 3:00 PM

Duration of exam:

2 hours 30 minutes

Number of exam pages:

14 (includes cover page)

Exam type:

Closed book

Materials allowed:

No additional materials are allowed


Additional instructions:

1. Complete the identity section above.


2. Write your answers in the space provided. If you need more room, use pages 13 - 14, and indicate this
on the question.
3. Do NOT detach any paper from this exam.
4. Your writing needs to be legible. Your arguments must be logical, clear and easy to understand.
5. There is a Proposition Sheet page provided separately from this exam booklet. There you will find some
of the major propositions that were covered in class. You may use any result from the proposition sheet
without proof. Make sure to clearly state the name or the acronym of the result you are using.
6. There are 13 questions. The total marks available in this exam is 50.

For each of Questions 1 to 4, full marks will be given for a correct answer which is placed in the box
provided. Part marks will only be awarded if relevant work is shown in the space provided.
1. Find integers x and y so that 27x + 35y = 1.

[3 marks]

Solution: Use the Extended Euclidean Algorithm:


x

35

27

13

10

We find that gcd(27, 35) = 1, so a solution exists by the Linear Diophantine Equation Theorem 1. Moreover,
by the second to last row of the table above, we have 27(13) + 35(10) = 1, so
x = 13, y = 10.
2. Find all complex number solutions to the equation x4 16 = 0. Give your answers in standard form.
[2 marks]
Solution: Rearrange the equation to obtain x4 = 16. We therefore have to find the complex fourth roots
of 16. As |16| = 16 and arg(16) = 0, by the Complex nth Roots Theorem, the fourth roots of 16 are given by
!
!!
!
!!

2k
2k
k
k
4
x = 16 cos
+ i sin
= 2 cos
+ i sin
,
4
4
2
2
for k {0, 1, 2, 3}. Substituting in these values of k, we obtain the solutions
x { 2, 2i, 2, 2i }.

3. Find real numbers


r 0 and 0 < 2 such that 1

number 1 3i)

3i = rei (this is the exponential form of the complex


[2 marks]

Solution: Let z = 1 3i. Write z = rei = r(cos + i sin ), by definition of the complex exponential. But
this is just the polar form of a complex number, so we see that r = |z| and = arg(z). We therefore have



r = 1 3i = 1 + 3 = 2,
 

= arg(1 3i) = arctan 3 .


 

5
Now, in the range specified for , arctan 3 takes the two values 2
3 and 3 . But 1 3i is in the lower
right quadrant of the complex plane, so the correct value must be = 5
3 . We obtain
r = 2, =

5
3 .

4. Find all solutions to the equation x2 = [1] in Z5 .

[2 marks]

Solution: There are only 5 congruence classes in Z5 , so we can simply test them all:
x Z5

x2 Z5

[0]

[0]

[1]

[1]

[2]

[4]

[3]

[9] = [4]

[4]

[16] = [1]

Thus, we see that the solutions to the equation x2 = [1] in Z5 are


x { [1], [4] }

For each part of Question 5 in the box provided, indicate whether the given statement is true (T) or false (F).
No justification is required.
1.5 marks will be given for each correct answer.
0.5 marks will be deducted for each incorrect answer.
If you choose not to answer a part (i.e. you leave the box for that part empty), you will receive 0.5
marks for that part.
The minimum mark for this question will be 0.
5.

[9 marks]

(a) The negation of A B is A (B).


FALSE. The correct negation of A B is A (B).
(b) The polynomial x3 3x2 + x + 2 has a rational root.
p

TRUE. By the Rational Roots Theorem, any rational root x = q must have p | 2 and q | 1. The only
possibilities are x = 1 and x = 2. Testing them all, we see that x = 2 is a root.
(c) 75 is a valid n in the RSA encryption scheme.
FALSE. In RSA, n = pq for a pair of prime numbers p and q. But 75 is not a product of two primes
(in fact, 75 = 3 5 5), so it is not a valid n.
(d) If f (z) is a polynomial with complex coefficients and z = i is a root of f (z), then z = i must also be a
root of f (z).
FALSE. This statement only holds if the coefficients of f (z) are all real numbers. A simple counterexample in the complex coefficients case is f (z) = z i.
(e) A B is logically equivalent to (B) (A).
TRUE. This is the contrapositive.
(f) 1 + i is an eighth root of unity.
FALSE. The complex modulus of 1 + i is
unity must have modulus 1).

2, so it cannot possibly be a root of unity (all roots of

Questions 6 and 7 involve more computation.


For full marks, you must show work that justifies your final answer.
6. Find all solutions z C to the equation z =

1+8i
2z .

Express your answers in standard form.

[5 marks]

Solution: Multiply both sides by (2 z) to obtain


z(2 z) = 1 + 8i
and rearrange to get
z2 2z + 1 + 8i = 0.
This is a quadratic, so we may solve it using the quadratic formula:

2 32i
z=
= 1 8i = 1 2 2i.
2
To find z, we therefore have to compute the square roots of 2i. In polar form, we have

 
 
3
3
+ i sin
2i = 2 cos
2
2
so, by the Complex nth Roots Theorem, the square roots of 2i are given by





3
3
2 cos
+ k + i sin
+ k
4
4
for k {0, 1}. Plugging in these values of k, we find that the square roots of 2i are
!

1
1
2 + i = 1 + i
and
2
2
!

1
1
2 i = 1 i.
2
2
Substituting back into the formula for z, we find that z is either 1 + 2(1 i) = 3 2i or 1 2(1 i) = 1 + 2i.
i.e.
z {3 2i, 1 + 2i}.

7.

(a) Find the prime factorization of gcd(5! , 2880).


Solution: The prime factorization of 5! is computed as

[2 marks]

5! = 1 2 3 4 5 = 23 3 5.
The prime factorization of 2880 is computed as
2880 = 2 1440 = 22 720 = 23 360 = 24 180 = 25 90 = 26 45 = 26 32 5.
By GCD by Prime Factorization, the prime factorization of gcd(5! , 2880) is therefore
gcd(5! , 2880) = 23 3 5.

(b) How many positive divisors does gcd(5! , 2880) have?


[2 marks]
Solution: We can compute the positive divisors of gcd(5! , 2880) from its prime factorization, using
Divisors from Prime Factorization. We obtain:
20 30 50

21 30 50

22 30 50

23 30 50

20 31 50

21 31 50

22 31 50

23 31 50

20 30 51

21 30 51

22 30 51

23 30 51

20 31 51

21 31 51

22 31 51

23 31 51

Thus, gcd(5! , 2880) has 16 positive divisors.

The remaining questions require you to write proofs. Write clearly and justify your steps.
Do NOT use the amount of space provided as an indication of how long your answer should be.
8. Let p be a prime and a be an integer such that p - a. Consider the two sets
S = {x N | ax 1 (mod p) },
T = {n(p 1) | n N }.
Prove that T S.

[2 marks]

Solution: Let x be any element of T. Then x = n(p 1) for some n N. We have to show that x S. To
do this, it suffices to show that ax = an(p1) 1 (mod p).
As p - a, by Fermats Little Theorem, we have that
ap1 1
So

(mod p).


n
ax = an(p1) = ap1 1n 1

Thus we see that x S, and therefore T S.

(mod p).


9. Let p be any prime number. Prove that the linear congruence


px 6

(mod 18)

has a solution x Z.

[3 marks]

Solution: Let p be prime and let d = gcd(p, 18). By the Linear Congruence Theorem, the linear congruence
px 6

(mod 18)

has a solution x Z if and only if d | 6.


We consider three cases:
If p = 2, then d = 2 and so, since 2 | 6, the linear congruence has an integer solution.
If p = 3, then d = 3 and so, since 3 | 6, the linear congruence has an integer solution.
If p > 3 then p - 18 (as p is prime and the largest prime factor of 18 is 3). So d , p in this case, which
means that we must have d = 1 (as the only positive divisors of p are 1 and itself). Since 1 | 18, the
linear congruence has an integer solution.
Thus we see that, for any prime p, the linear congruence
px 6
has a solution x Z.

(mod 18)


10. Prove the following proposition:


Proposition 1. Let p, q N be coprime and let x be an integer satisfying
xp

(mod q)

x q (mod p)
then x p + q (mod pq).

[3 marks]

Solution: Note first that x0 = p + q is an integer solution to the simultaneous linear congruences
xp

(mod q)

x q (mod p)
since x0 = p + q p (mod q) and x0 = p + q q (mod p).
To find the complete solution, we use the Chinese Remainder Theorem. As p and q are coprime, we have
gcd(p, q) = 1 by definition, so the Chinese Remainder Theorem tells us that the complete solution to the
simultaneous linear congruences above is
x p + q (mod pq)
as required.

11. Let c = a + bi be a complex number in standard form, with b , 0, and let f (x) C[x] be a polynomial.
Suppose further that all of the coefficients of f (x) are real numbers. Prove that f (c) = 0 if and only if
(x2 2ax + a2 + b2 ) is a factor of f (x).
[5 marks]
Solution: () Begin by assuming that f (c) = 0. Then, by the Factor Theorem, (x c) divides f (x). Thus,
we may write
f (x) = (x c)g(x)
for some g(x) C[x].
Now, since the coefficients of f (x) are all real numbers, by the Conjugate Roots Theorem, we also have
f (c) = 0. Substitute x = c into the equation above to obtain
f (c) = (c c)g(c) = 0.
As b , 0, we have c , c, so c c , 0 and therefore we must have g(c) = 0. So, by the Factor Theorem again,
(x c) divides g(x) in C[x] and we can write
f (x) = (x c)(x c)h(x)
for some h(x) C[x].
Now,
(x c)(x c) = (x a bi)(x a + bi) = x2 2ax + a2 + b2 ,
so (x2 2ax + a2 + b2 ) is a factor of f (x).
() Next assume that (x2 2ax + a2 + b2 ) is a factor of f (x). Then
f (x) = (x2 2ax + a2 + b2 )g(x)
for some g(x) C[x]. Plug in x = c = a + bi to obtain
f (c) = (c2 2ac + a2 + b2 )g(c) = (a2 + 2abi b2 2a2 2abi + a2 + b2 )g(c) = 0.


12. Let p and q be distinct odd primes. Prove that gcd(p + q, p q) = 2.

[5 marks]

Solution: Let d = gcd(p + q, p q). Since both p and q are odd primes, p + q and p q are both even
numbers, so 2 | p + q and 2 | p q. Therefore d 2.
Now, by definition, d | (p + q) and d | (p q). So, by Divisibility of Integer Combinations
d | (p + q) + (p q)

d | 2p

d | (p + q) (p q)

d | 2q.

and
Moreover, as p and q are distinct, gcd(p, q) = 1. So, by the Extended Euclidean Algorithm, there exist integers
x and y such that
px + qy = 1.
Multiply this equation by 2 to obtain
2px + 2qy = 2.
Now, since d | 2p and d | 2q, by Divisibility of Integer Combinations again, we have
d | (2px + 2qy)

d | 2.

From this, Bounds by Divisibility gives d 2. Thus, as d 2 and d 2, we must have d = 2.

13. Prove that there are no integers x and y that satisfy x2 + 3y2 = 8.

[5 marks]

Solution: Suppose, for a contradiction, that an integer solution (x, y) to x2 + 3y2 = 8 exists. Let u = x2
and v = y2 . Then (u, v) is a solution to the linear Diophantine equation
u + 3v = 8.
This equation has the obvious particular solution u0 = 8, v0 = 0. Since gcd(1, 3) = 1, by the Linear
Diophantine Equation Theorem 2, the solution (u, v) = (x2 , y2 ) must be given by
x2 = 8 + 3n
y2 = n
for some n Z.
Now, x2 0 and y2 0, so we must have n 38 and n 0. The only possibilities are therefore
n {2, 1, 0}. Substituting in, we obtain
(x2 , y2 ) = (8, 0), (5, 1), or (2, 2).
In other words, x2 is either 2, 5 or 8. But none of 2, 5 or 8 is a perfect square. This is a contradiction, so
no integer solution (x, y) to x2 + 3y2 = 8 exists.


This page has been left blank for rough or overflow work.
Do not tear this page off.

This page has been left blank for rough or overflow work.
Do not tear this page off.

You might also like